Question

In: Finance

Consider a world that only consists of the three stocks shown in the following​ table: LOADING......

Consider a world that only consists of the three stocks shown in the following​ table:

LOADING...

.

a. Calculate the total value of all shares outstanding currently.

b. What fraction of the total value outstanding does each stock make​ up?

c. You hold the market​ portfolio, that​ is, you have picked portfolio weights equal to the answer to part b with each​ stock's weight is equal to its contribution to the fraction of the total value of all stocks. What is the expected return of your​ portfolio?

Stock

Total Number

of Shares Outstanding

Current Price per Share

Expected Return

First Bank

101101 million

​$8282

2020​%

Fast Mover

5959 million

​$132132

1313​%

Funny Bone

190190 million

​$3232

1414​%

Solutions

Expert Solution

First Bank Fast Mover Funny Bone
Shares outstanding 101000000 59000000 190000000
Price per share 82 132 32
Expected return 20% 13% 14%
a Value of shares 8282000000 7788000000 6080000000
Total value 22150000000
b Fraction 0.3739 0.3516 0.2745
c Expected return 15.89%

WORKINGS


Related Solutions

Consider a world that only consists of the three stocks shown in the following​ table: LOADING.......
Consider a world that only consists of the three stocks shown in the following​ table: LOADING.... a. Calculate the total value of all shares outstanding currently. b. What fraction of the total value outstanding does each stock make​ up? c. You hold the market​ portfolio, that​ is, you have picked portfolio weights equal to the answer to part b with each​ stock's weight is equal to its contribution to the fraction of the total value of all stocks. What is...
 For the project shown in the following​ table, LOADING... ​, calculate the internal rate of return...
 For the project shown in the following​ table, LOADING... ​, calculate the internal rate of return ​(IRR). Then​ indicate, for the​ project, the maximum cost of capital that the firm could have and still find the IRR acceptable. Initial investment ​(CF 0CF0​) ​$160 comma 000160,000 Copy to Clipboard + Open in Excel + Year ​(t​) Cash inflows ​(CF Subscript tCFt​) 1 ​$45 comma 00045,000 2 ​$50 comma 00050,000 3 ​$45 comma 00045,000 4 ​$30 comma 00030,000 5 ​$35 comma 00035,000...
Consider the following​ stocks, LOADING... ​, all of which will pay a liquidating dividend one year...
Consider the following​ stocks, LOADING... ​, all of which will pay a liquidating dividend one year from now and nothing in the interim. Assume the​ risk-free rate is 4 % and the market risk premium is 7 % a. What does the CAPM predict the expected return for each stock should​ be? b. ​Clearly, the CAPM predictions are not equal to the actual expected​ returns, so the CAPM does not hold. You decide to investigate this further. To see what...
Consider the three stocks in the following table. Pt represents price at time t, and Qt...
Consider the three stocks in the following table. Pt represents price at time t, and Qt represents shares outstanding at time t. Stock C splits two for one in the last period. P0 Q0 P1 Q1 P2 Q2 A 90 100 95 100 95 100 B 50 200 45 200 45 200 C 100 200 110 200 55 400 a. Calculate the rate of return on a price-weighted index of the three stocks for the first period ( t 5...
Consider the three stocks in the following table. Pt represents price at time t, and Qt...
Consider the three stocks in the following table. Pt represents price at time t, and Qt represents shares outstanding at time t. Stock C splits two-for-one in the last period. P0 Q0 P1 Q1 P2 Q2 A 100 100 105 100 105 100 B 60 200 55 200 55 200 C 120 200 130 200 65 400 Calculate the first-period rates of return on the following indexes of the three stocks: (Do not round intermediate calculations. Round your answers to...
Consider the three stocks in the following table. Ptrepresents price at time t, and Qtrepresents shares...
Consider the three stocks in the following table. Ptrepresents price at time t, and Qtrepresents shares outstanding at time t. There are three time points 0, 1, and 2. P0 Q0 P1 Q1 P2 Q2 A 90 100 95 100 100 100 B 50 200 45 200 45 200 C 100 200 110 200 105 200 a) Form an equally weighted portfolio in stocks A, B, C at time 0 and hold it to time 1. Calculate the portfolio return...
Consider the three stocks in the following table. Pt represents price at time t, and Qt...
Consider the three stocks in the following table. Pt represents price at time t, and Qt represents shares outstanding at time t.            Po      Q0        P1        Q1 A         87      100       92      100 B        47      200     42       200 C        94      200      104    200 What is the rate of return on the equal-weighted index of the three stocks? O 1.92% O 10.64% O. 4.07% O 4.39%
Consider the three stocks in the following table. Pt represents price at time t, and Qt...
Consider the three stocks in the following table. Pt represents price at time t, and Qt represents shares outstanding at time t. Stock C splits two-for-one in the last period. P0 Q0 P1 Q1 P2 Q2 A 80 100 85 100 85 100 B 40 200 35 200 35 200 C 80 200 90 200 45 400 Calculate the first-period rates of return on the following indexes of the three stocks: (Do not round intermediate calculations. Round your answers to...
Consider the three stocks in the following table. Pt represents price at time t, and Qt...
Consider the three stocks in the following table. Pt represents price at time t, and Qt represents shares outstanding at time t. Stock C splits two-for-one in the last period. P0 Q0 P1 Q1 P2 Q2 A 85 100 90 100 90 100 B 45 200 40 200 40 200 C 90 200 100 200 50 400 a. Calculate the rate of return on a price-weighted index of the three stocks for the first period (t = 0 to t...
Consider the three stocks in the following table. Pt represents price at time t, and Qt...
Consider the three stocks in the following table. Pt represents price at time t, and Qt represents shares outstanding at time t. Stock C splits two-for-one in the last period. P0 Q0 P1 Q1 P2 Q2 A 84 100 89 100 89 100 B 44 200 39 200 39 200 C 88 200 98 200 49 400 a. Calculate the rate of return on a price-weighted index of the three stocks for the first period (t = 0 to t...
ADVERTISEMENT
ADVERTISEMENT
ADVERTISEMENT